LSAT and Law School Admissions Forum

Get expert LSAT preparation and law school admissions advice from PowerScore Test Preparation.

 Administrator
PowerScore Staff
  • PowerScore Staff
  • Posts: 8916
  • Joined: Feb 02, 2011
|
#26695
Please post below with any questions!
 lsatstudent
  • Posts: 7
  • Joined: May 29, 2016
|
#26837
Why is A correct? Is it because the author first says that the sample size is too small to draw a conclusion about and then proceeds to draw a conclusion?
User avatar
 Dave Killoran
PowerScore Staff
  • PowerScore Staff
  • Posts: 5853
  • Joined: Mar 25, 2011
|
#26839
Hi LSATStudent,

Thanks for the question! Yes, that is exactly correct—well done! The stimulus notes early on that "Scientifically valid inferences could not be drawn from the study because of the small sample size" but then later says that "This proves that..." These two positions contradict each other, which is the same thing as being inconsistent. Answer choice (A) describes that problem that perfectly.

Please let me know if that helps. Thanks!
 tamarisk
  • Posts: 4
  • Joined: Sep 15, 2016
|
#28904
I understand that A is correct, but what about D? Even though A is biggest flaw when it comes to the answers, is D considered a actual flaw of the argument or is the answer actually just plain wrong? When I try to switch the chemical exposure and banning in my head back and forth as the cause and effect based on what is written in the stimulus, it seems to be legitimate in saying that the author made that kind of error.
User avatar
 Jonathan Evans
PowerScore Staff
  • PowerScore Staff
  • Posts: 726
  • Joined: Jun 09, 2016
|
#29113
Hi, Tamarisk,

On flaw questions, if you make an answer choice a contender, you have to match that answer choice exactly to corresponding concepts in the stimulus. To be certain that you have the answer right, there is no room for settling for "good enough." You must have an affirmative reason why the answer you choose is correct.

Take answer choice D. Isolate the concepts in the answer choice you wish to match. First what would one posit is the "something" that is the "cause of the reduction?" The only plausible match is the "regulations banning PCB." Now ask could the "regulations" be an effect of the reduction? Clearly not. There is no match, so in this case answer choice D does not describe a flaw here.

Good question. I hope this helps.

Get the most out of your LSAT Prep Plus subscription.

Analyze and track your performance with our Testing and Analytics Package.